数分笔记——Fejér-Jackson不等式

【注:待更一些更强的结论】

参考书:梅加强《数学分析》. 下面文章也可以参考:

题目:若数列 \{na_n\} 单调收敛于0, 则函数项级数 \sum\limits_{n=1}^{\infty}a_n\sin nx 在 \mathbb{R} 中一致收敛.

证明:根据Dirichlet判别法, 我们只需要证明级数 \sum\limits_{n=1}^{\infty}\dfrac{\sin nx}{n} 一致有界. \QED

所以问题的关键在于,怎么证级数 \sum\limits_{n=1}^{\infty}\dfrac{\sin nx}{n} 一致有界?关于这个问题可以引伸出一个更有意思的问题:它由Fejér于1910年猜想, 且由Fejér,Jackson,Gronwall等人首先证明:

定理1 [Fejér-Jackson不等式] 设 x\in(0,\pi), 则必有 \sum\limits_{k=1}^n\dfrac{\sin kx}{k}>0.

证明[1]:记f_n(x)=\sum\limits_{k=1}^n\dfrac{\sin kx}{k}, 则

\begin{aligned} f_n'(x)&=\sum\limits_{k=1}^n\cos kx \\ &=\mathrm{Re}\sum\limits_{k=1}^n(e^{ix})^k \\ &=\mathrm{Re}\left(e^{ix}\dfrac{e^{inx}-1}{e^{ix}-1}\right) \\ &=\dfrac{\cos\frac{(n+1)x}{2}\sin\frac{nx}{2}}{\sin\frac{x}{2}}. \end{aligned}\\

于是, f_n'(x)的所有零点满足

\dfrac{nx}{2}=k\pi(k\in\mathbb{Z}) \\

以及

\dfrac{(n+1)x}{2}=\dfrac{\pi}{2}+k\pi(k\in\mathbb{Z}). \\

(0,\pi)上, 这些零点从小到大排列为

0 < \dfrac{\pi}{n+1} < \dfrac{2\pi}{n} < \dfrac{3\pi}{n+1} < \dfrac{4\pi}{n} < \cdots < \dfrac{2\lfloor n/2\rfloor\pi}{n}\le \pi. \\

(0,\dfrac{\pi}{n+1})上, f_n'(x)>0. 于是, 从0开始往x轴正方向, 每经过一个零点, f_n'(x)的符号就变化一次. 从而

\dfrac{2\pi}{n}, \dfrac{4\pi}{n},\cdots,\dfrac{2\lfloor n/2\rfloor\pi}{n} \\

f_n(x)的所有极小值点.

下面用数学归纳法来证明f_n(x)>0在(0,\pi)上成立.

(i)首先当n=1时f_1(x)=\sin x, 结论显然成立.

(ii)下面假设f_{n-1}(x)>0在(0,\pi)上成立(其中n\ge 2).

注意到, 对j=1,2,\cdots,\lfloor n/2\rfloor,

f_n\left(\dfrac{2j\pi}{n}\right) =f_{n-1}\left(\dfrac{2j\pi}{n}\right) + \sin \left(n\cdot\dfrac{2j\pi}{n}\right) = f_{n-1}\left(\dfrac{2j\pi}{n}\right) >0,\\

最后的不等号是因为归纳假设. 于是, f_n(x)在所有极小值点处都大于0, 从而f_n(x)>0在(0,\pi)上恒成立. \square

 

注:在百度贴吧找到一个更好的结果(未验证正确性):

命题3 对任意 x\in(0,\pi),n\in\mathbb{N}^* 都有 \sum\limits_{k=1}^n\dfrac{\sin kx}{k}>x\left(1-\dfrac{x}{\pi}\right)^3.
 
定理4[一致有界性] 设 x\in\mathbb{R}, 则存在与 x 无关的常数 M>0 使得
\left|\sum\limits_{k=1}^n\dfrac{\sin kx}{k}\right|<M,\forall n\in\mathbb{N}^*,\forall x.

Motivation: 回顾不等式 \sin t\leq t(t\geq 0) 以及

\begin{aligned} \sum\limits_{k=m+1}^n\sin kx&=\dfrac{1}{2\sin\frac{x}{2}}\sum\limits_{k=m+1}^n \left[\cos\left(k+\dfrac{1}{2}\right)x-\cos\left(k-\dfrac{1}{2}\right)x\right] \\ &=\dfrac{\cos\left(m+\dfrac{1}{2}\right)x-\cos\left(n+\dfrac{1}{2}\right)x}{2\sin\frac{x}{2}},\\ \Rightarrow \left|\sum\limits_{k=m+1}^n\sin kx\right|&\leq\dfrac{1}{\left|\sin\frac{x}{2}\right|} \end{aligned}

注意 \sin t\leq t 在t比较小的范围内是比较“好”的, 而当t比较大的时候直接用三角函数有界性即可放缩. 所以当t比较大的时候, 可以考虑采用Abel变换以及三角级数的求和.

证明:由三角函数的周期性, 只需要考虑 (-\pi,\pi) 区间. 又由 \sin kx 是奇函数, 故只需要考虑 (0,\pi) 区间(另一半区间相当于取相反数). 【下面的证明在@博雅 的提醒下作了修改.】

(1)若 n\leq\dfrac{\sqrt{2\pi}}{x}, 根据不等式 \sin t<t(t>0), 得

\sum\limits_{k=1}^n\dfrac{\sin kx}{k} \leq\sum\limits_{k=1}^n\dfrac{kx}{k}=nx\leq\sqrt{2\pi}<2\sqrt{2\pi}.

(2)若 n>\dfrac{\sqrt{2\pi}}{x}, 取正整数 m 满足 mx<\sqrt{2\pi}<(m+1)x, 则 m<n. 把级数拆成两个部分:

\sum\limits_{k=1}^n\dfrac{\sin kx}{k}=\sum\limits_{k=1}^m\dfrac{\sin kx}{k} +\sum\limits_{k=m+1}^n\dfrac{\sin kx}{k}.

根据前一定理, 我们在这里不需要加绝对值.

由不等式 \sin t<t(t>0), 得

\sum\limits_{k=1}^m\dfrac{\sin kx}{k} \leq\sum\limits_{k=1}^m\dfrac{kx}{k}=mx.

回顾Abel求和, 设 S_n=\sum\limits_{k=1}^n\sin kx, 则

\begin{aligned} \sum\limits_{k=m+1}^n\dfrac{\sin kx}{k} &= \sum\limits_{k=m+1}^n\dfrac{S_k}{k}-\sum\limits_{k=m+1}^n\dfrac{S_{k-1}}{k} \\ &=\sum\limits_{k=m+1}^n\dfrac{S_k}{k}-\sum\limits_{k=m}^{n-1}\dfrac{S_{k}}{k+1} \\ &=\left[\sum\limits_{k=m+1}^{n-1}S_k\left(\dfrac{1}{k}-\dfrac{1}{k+1}\right)\right] +\dfrac{S_n}{n}-\dfrac{S_m}{m+1}. \end{aligned}

根据 x\in(0,\pi) 以及前一定理可得

\begin{aligned} \sum\limits_{k=m+1}^n\dfrac{\sin kx}{k} &\leq\sum\limits_{k=m+1}^{n-1}S_k\left(\dfrac{1}{k}-\dfrac{1}{k+1}\right) +\dfrac{S_n}{n}-\dfrac{S_m}{m+1} \\ &\leq \dfrac{1}{\sin\frac{x}{2}}\left(\dfrac{1}{m+1}-\dfrac{1}{n}\right) +\dfrac{1}{\sin\frac{x}{2}}\cdot\left(\dfrac{1}{n}+\dfrac{1}{m+1}\right) \\ &=\dfrac{2}{(m+1)\sin\frac{x}{2}}. \end{aligned}

注意当 x\in\left(0,\frac{\pi}{2}\right) 时有 \sin x>\dfrac{2x}{\pi}, 所以

\sum\limits_{k=m+1}^n\dfrac{\sin kx}{k}\leq \dfrac{2}{(m+1)\sin\frac{x}{2}}<\dfrac{2\pi}{(m+1)x}.

综上,

\sum\limits_{k=1}^n\dfrac{\sin kx}{k}<mx+\dfrac{2\pi}{(m+1)x} <\sqrt{2\pi}+\dfrac{2\pi}{\sqrt{2\pi}}=2\sqrt{2\pi}.

这里正整数m是待定的(与x有关), 取 mx<\sqrt{2\pi}<(m+1)x, 则上界 M 可以取为 2\sqrt{2\pi}. \QED


【原回答】

把级数拆成两个部分: (m待定)

\sum\limits_{k=1}^n\dfrac{\sin kx}{k}=\sum\limits_{k=1}^m\dfrac{\sin kx}{k} +\sum\limits_{k=m+1}^n\dfrac{\sin kx}{k}.

如果 m<1, 则第一部分为0, 如果 m>n, 则第二部分为0. 根据定理1, 我们在这里不需要加绝对值.

(1)根据不等式 \sin t<t(t>0), 得

\sum\limits_{k=1}^m\dfrac{\sin kx}{k} \leq\sum\limits_{k=1}^m\dfrac{kx}{k}=mx.

(2)回顾Abel求和, 设 S_n=\sum\limits_{k=1}^n\sin kx, 则

\begin{aligned} \sum\limits_{k=m+1}^n\dfrac{\sin kx}{k} &= \sum\limits_{k=m+1}^n\dfrac{S_k}{k}-\sum\limits_{k=m+1}^n\dfrac{S_{k-1}}{k} \\ &=\sum\limits_{k=m+1}^n\dfrac{S_k}{k}-\sum\limits_{k=m}^{n-1}\dfrac{S_{k}}{k+1} \\ &=\left[\sum\limits_{k=m+1}^{n-1}S_k\left(\dfrac{1}{k}-\dfrac{1}{k+1}\right)\right] +\dfrac{S_n}{n}-\dfrac{S_m}{m+1}. \end{aligned}

根据 x\in(0,\pi) 可得

\begin{aligned} \sum\limits_{k=m+1}^n\dfrac{\sin kx}{k} &\leq\sum\limits_{k=m+1}^{n-1}S_k\left(\dfrac{1}{k}-\dfrac{1}{k+1}\right) +\dfrac{S_n}{n}-\dfrac{S_m}{m+1} \\ &\leq \dfrac{1}{\sin\frac{x}{2}}\left(\dfrac{1}{m+1}-\dfrac{1}{n}\right) +\dfrac{1}{\sin\frac{x}{2}}\cdot\left(\dfrac{1}{n}+\dfrac{1}{m+1}\right) \\ &=\dfrac{2}{(m+1)\sin\frac{x}{2}}. \end{aligned}

这里还需要处理 \sin\dfrac{x}{2}. 注意当 x\in\left(0,\frac{\pi}{2}\right) 时有 \sin x>\dfrac{2x}{\pi}, 所以

\sum\limits_{k=m+1}^n\dfrac{\sin kx}{k}\leq \dfrac{2}{(m+1)\sin\frac{x}{2}}<\dfrac{2\pi}{(m+1)x}.

综上,

\sum\limits_{k=1}^n\dfrac{\sin kx}{k}<mx+\dfrac{2\pi}{(m+1)x}.

这里正整数m是待定的(与x有关), 取 mx<\sqrt{2\pi}<(m+1)x, 则上界 M 可以取为 2\sqrt{2\pi}. \QED

【原回答over】


注:稍微修改某一步, 可以把上界 M 变小为 2\sqrt{\pi}.

注:当 x\in\left(0,\frac{\pi}{2}\right) 时有 \sin x>\dfrac{2x}{\pi}, 这个不等式非常实用,例如:

(1)(用围道积分)计算Fresnel积分 \int_0^{\infty}\sin x^2dx=\int_0^{\infty}\cos x^2dx=\dfrac{\sqrt{2\pi}}{4}.
(2)(用留数定理)计算 \int_0^{+\infty}\dfrac{x\sin x}{x^2+1}dx.
(3)证明 \dfrac{\pi}{2\sqrt{2}}<\int_0^1\dfrac{dx}{\sqrt{1-x^4}}<\dfrac{\pi}{2}\ln(1+\sqrt{2}).

更多估计:

定理5 设 0<x<\pi, 则 \sum\limits_{k=1}^n\dfrac{\sin kx}{k}<\int_0^{\pi}\dfrac{\sin x}{x}dx =1.8519\cdots.

证明:设 f_n(x)=\sum\limits_{k=1}^n\dfrac{\sin kx}{k}. 则 f_n(0)=f_n(\pi)=0, 且

\begin{aligned} f_n'(x)&=\sum\limits_{k=1}^n\cos kx=\dfrac{\cos\dfrac{(n+1)x}{2}\sin\dfrac{nx}{2}}{\sin x}\\ &=\dfrac{\sin\left(nx+\dfrac{x}{2}\right)}{2\sin\dfrac{x}{2}} -\dfrac{1}{2} =\dfrac{\sin (n+1)x}{2\tan\dfrac{x}{2}}-\dfrac{1}{2}(\cos (n+1)x+1) \leq\dfrac{\sin (n+1)x}{2\tan\dfrac{x}{2}} . \end{aligned}

且 f_n'(x)=\dfrac{\sin nx}{2\tan\dfrac{x}{2}}+\dfrac{1}{2}(\cos nx-1) \leq\dfrac{\sin nx}{2\tan\dfrac{x}{2}} .

令 f_n'(x)=0, 则 \dfrac{n+1}{2}x=j\pi-\dfrac{\pi}{2}(j=1,2,\cdots,\lfloor n/2\rfloor) 或者 \dfrac{nx}{2}=j\pi, (j=1,\cdots,\lfloor n/2\rfloor) (这里 j=0 时 f(0)=0, 为平凡的情况). 即 x=\dfrac{2j-1}{n+1}\pi 或 x=\dfrac{2j\pi}{n}, (j=1,2,\cdots,\lfloor n/2\rfloor). 记 x_j=\dfrac{2j-1}{n+1}\pi, y_j=\dfrac{2j\pi}{n}. 由于

\begin{aligned} &\,\,\,\,\,\,\, f_n(x_{j+1})-f_n(x_j) \\ &=\int_{x_j}^{x_{j+1}}f'(t)dt \leq\int_{x_j}^{x_{j+1}}\dfrac{\sin (n+1)t}{2\tan\dfrac{t}{2}}dt \\ &=\int_{(2j-1)\pi}^{(2j+1)\pi}\dfrac{\sin t}{2(n+1)\tan\dfrac{t}{2(n+1)}}dt \text{(拆成两段)}\\ &=\int_{2j\pi}^{(2j+1)\pi}\dfrac{\sin t}{2(n+1)\tan\dfrac{t}{2(n+1)}}dt +\int_{2j\pi}^{(2j+1)\pi}\dfrac{\sin (t+\pi)}{2(n+1)\tan\dfrac{t+\pi}{2(n+1)}}dt \\ &=\int_{2j\pi}^{(2j+1)\pi}\dfrac{\sin t}{2(n+1)} \left(\dfrac{1}{\tan\dfrac{t}{2(n+1)}}-\dfrac{1}{\tan\dfrac{t+\pi}{2(n+1)}}\right)dt \end{aligned}

函数 \dfrac{1}{\tan x} 在区间 (0,\pi/2) 单调递减, 且 j<\dfrac{n}{2}, 则 \dfrac{1}{\tan\dfrac{t}{2(n+1)}}-\dfrac{1}{\tan\dfrac{t+\pi}{2(n+1)}}<0. 另外当 t\in(2j\pi, (2j+1)\pi) 时 \sin t>0, 所以 f_n(x_{j+1})<f_n(x_j).

同理可得f_n(y_{j+1})<f_n(y_j), 因此

\max_{x\in[0,\pi]}f_n(x)=\max\{f_n(x_1),f_n(y_1)\}.

下面设 a_n=f_n\left(\dfrac{\pi}{n+1}\right)=\sum\limits_{k=1}^n\dfrac{1}{k}\sin \dfrac{k\pi}{n+1}, 则

\begin{aligned} a_{n+1}&=f_{n+1}\left(\dfrac{\pi}{n+2}\right) >f_{n+1}\left(\dfrac{\pi}{n+1}\right) \\ &=f_{n}\left(\dfrac{\pi}{n+1}\right)+\dfrac{1}{n+1}\sin(n+1)\dfrac{\pi}{n+1} \\ &=f_{n}\left(\dfrac{\pi}{n+1}\right)=a_n. \end{aligned}

所以数列 \{a_n\} 单调递增, 且有上界(前面已经证明了有 2\sqrt{2\pi} 为上界), 因此极限存在.

\begin{aligned} \lim\limits_{n\to\infty}a_n &=\lim\limits_{n\to\infty}\sum\limits_{k=1}^{n+1}\dfrac{1}{k}\sin\dfrac{k\pi}{n+1}\\ &=\lim\limits_{n\to\infty}\dfrac{1}{n+1}\sum\limits_{k=1}^{n+1}\dfrac{n+1}{k}\sin\dfrac{k\pi}{n+1}\\ &=\int_0^1\dfrac{\sin \pi x}{x}dx=\int_0^{\pi}\dfrac{\sin t}{t}dt. \end{aligned}

(对 y_j 的验证是同理的)

因此 f_n(x)<\int_0^{\pi}\dfrac{\sin t}{t}dt, \forall n, \forall x\in(0,\pi). \QED

参考

  1. ^Fejer-Jackson不等式证明 https://math.stackexchange.com/questions/376273/